LSAT and Law School Admissions Forum

Get expert LSAT preparation and law school admissions advice from PowerScore Test Preparation.

 Administrator
PowerScore Staff
  • PowerScore Staff
  • Posts: 8917
  • Joined: Feb 02, 2011
|
#27344
Complete Question Explanation

Justify the Conclusion—PR. The correct answer choice is (D)

In this stimulus, the author defends the scientists whose discoveries were useful but had environmentally damaging consequences as well:
  • Premise: ..... The scientists merely generated the theories without being able to foresee nor restrict the resulting ..... ..... ..... products.

    Conclusion: ..... The researchers should not be held liable for the environmental damage.
The questions requires that we find the principle that will justify this reasoning. It will likely provide that one is not responsible for the unforeseeable and uncontrollable outcomes of one’s work.

Answer choice (A): One major problem with this answer choice is the phrase “improperly used.” The stimulus provides that the research led to products that were useful but damaging. Since there is no mention of “misuse,” this answer choice cannot justify the author’s conclusion.

Answer choice (B): The scientists discussed in this stimulus do not bear the undesirable consequences, so this answer choice is not relevant to the author’s argument, and certainly does not justify the conclusion.

Answer choice (C): The author is not concerned with the question of whether the scientists will receive credit—the focus is on whether they should be held responsible for the detrimental outcome of their research.

Answer choice (D): This is the correct answer choice—the one that completely justifies the conclusion from the stimulus, and basically restates our prephrased principle from the discussion above. If one is not responsible for the outcomes of one’s work which are both unforeseeable and beyond one’s control, then the scientists discussed in the stimulus should not be help liable for the detrimental outcomes of their work—the author specifically provides that the scientists could neither have foreseen nor controlled the detriments which followed.

Answer choice (E): Since the author’s conclusion is that the scientists shouldn’t be held responsible, this choice which asserts the opposite cannot possibly help to justify the author’s conclusion. Further, the stimulus makes no mention of getting credit for foreseeable positive outcomes.

Get the most out of your LSAT Prep Plus subscription.

Analyze and track your performance with our Testing and Analytics Package.